Answered step by step
Verified Expert Solution
Link Copied!

Question

1 Approved Answer

Two rms, Firm 1 and Firm 2, compete by simultaneously choosing prices. Both rms sell an identical product for which each of 100 consumers has

image text in transcribed
Two rms, Firm 1 and Firm 2, compete by simultaneously choosing prices. Both rms sell an identical product for which each of 100 consumers has a maximum willingness to pay of $40. Each consumer will buy at most 1 unit, and will buy it from whichever firm charges the lowest price. If both firms set the same price, they share the market equally. Costs are given by ci(q,-) = 1661,. Because of government regulation, rms can only choose prices which are integer numbers, and they cannot price above $40. Answer the following: a) (0.25 point) If Firm 1 chooses p1 = 27, Firm 2's best response is to set what price? b) (0.25 point) If Firm 2 chooses the price determined in the previous question, Firm 1's best response is to choose what price? c) (1 point) If Firm 1 chooses p1 = 10, Firm 2's best response is a range of prices. What is the lowest price in this range

Step by Step Solution

There are 3 Steps involved in it

Step: 1

blur-text-image

Get Instant Access to Expert-Tailored Solutions

See step-by-step solutions with expert insights and AI powered tools for academic success

Step: 2

blur-text-image

Step: 3

blur-text-image

Ace Your Homework with AI

Get the answers you need in no time with our AI-driven, step-by-step assistance

Get Started

Recommended Textbook for

Experimental Econophysics Properties And Mechanisms Of Laboratory Markets

Authors: Ji Ping Huang

1st Edition

3662442345, 9783662442340

More Books

Students also viewed these Economics questions

Question

List three dimensions by which we can describe the self-concept.

Answered: 1 week ago

Question

Calculate the mesh current i1 and i2 in Fig. 3.92? . 4) 82 2,

Answered: 1 week ago

Question

If you were Akio, what would you do now?

Answered: 1 week ago